LSAT and Law School Admissions Forum

Get expert LSAT preparation and law school admissions advice from PowerScore Test Preparation.

User avatar
 Dave Killoran
PowerScore Staff
  • PowerScore Staff
  • Posts: 5852
  • Joined: Mar 25, 2011
|
#88201
Complete Question Explanation
(The complete setup for this game can be found here: lsat/viewtopic.php?f=171&p=88197#p88197)

The correct answer choice is (B)

The condition in this question establishes that S is not immediately above O. Because S cannot be immediately above R (from the first rule), and also cannot be immediately above L or M (from the third rule), S must be immediately above V, the only remaining variable.

Because R cannot be consecutive with S, and V, O, L, and M are all below S, no other variable other than S is available to be sixth. Thus, S must be sixth, and from the prior inference V must then be fifth.

With S and V placed as sixth and fifth, the primary uncertainty is the placement of R, O, L, and M. O, L, and M must still adhere to the sequence established in the second and third rules, creating the following scenario:

G3-Q14-d1.png

Because O must always be above the LM block, the LM block must always be 1-2 or 2-3, and thus using Hurdle the Uncertainty we can determine that L or M is always second, and O is always third or fourth:

G3-Q14-d2.png

R can be first, third, or fourth.

Answer choice (A) is incorrect because S must be immediately above V.

Answer choice (B) is the correct answer. R can be fourth, O can be third, M can be second, and L can be first.

Answer choice (C) is incorrect because from the second rule L is immediately below M.

Answer choice (D) is incorrect because L or M must be second.

Answer choice (E) is incorrect because S is the top layer.
You do not have the required permissions to view the files attached to this post.
 lsnewbie
  • Posts: 12
  • Joined: Aug 31, 2018
|
#59869
Hello PS,

I got this question wrong, and I'm not entirely sure I understand why B is the right answer. I drew out a few scenarios where it seems B would be the best answer, for example --

R
O
S/V
V/S M
L


R/V
V/R
O
M
L
S

R/V
V/R
O
S
M
L

But if O was to be the top most layer, it would seem that there is still a way for R and S to be separated without R being above O, in which case B wouldn't be the right answer --

O
R/V
V/R
M
L
S

Could you please explain to me why B is the correct answer and what it is I'm missing? I'm not sure if I'm even going about this the right way. Also, is there a better way to approach these kinds of problems without drawing out potential scenarios? I feel like my approach is too time consuming.

Thank you!
JT
 Lauren Hartfiel
PowerScore Staff
  • PowerScore Staff
  • Posts: 5
  • Joined: Sep 26, 2018
|
#59881
Hello JT!

You forgot the rule that orange is below strawberry. The only set up possible if there is a flavor between strawberry and orange is:

S
V
O/R
R/O
M
L

Also, because it is a could be true question B would still be correct. Your extra scenarios show you that other answers could be true, and that should make you go back and look at the rules because only one answer can be correct. Don't worry, misreading a rule is a common issue and one that often gets me. Just remember if your setups are showing more than one answer to be correct you have missed something in the setup. And, always read very carefully on the LSAT, this is the primary key to success! Let me know if you need further explanation :)

Get the most out of your LSAT Prep Plus subscription.

Analyze and track your performance with our Testing and Analytics Package.